Những câu hỏi liên quan
fghj
Xem chi tiết
phan thị minh anh
Xem chi tiết
Hoàng Lê Bảo Ngọc
10 tháng 9 2016 lúc 10:53

Ta có : \(\frac{1}{x}+\frac{1}{y}+\frac{1}{z}=1\Leftrightarrow xy+yz+zx=xyz\)

\(\sqrt{x+yz}+\sqrt{y+zx}+\sqrt{z+xy}\ge\sqrt{xyz}+\sqrt{x}+\sqrt{y}+\sqrt{z}\)

Bình phương vế trái : 

\(\left(\sqrt{x+yz}+\sqrt{y+zx}+\sqrt{z+xy}\right)^2\)

\(=\left(x+y+z+xy+yz+zx\right)+2\left(\sqrt{x+yz}.\sqrt{y+zx}+\sqrt{y+zx}.\sqrt{z+xy}+\sqrt{z+xy}.\sqrt{x+yz}\right)\)Bình phương vế phải : 

\(\left(\sqrt{xyz}+\sqrt{x}+\sqrt{y}+\sqrt{z}\right)^2=\left(xyz+x+y+z\right)+2\left(x\sqrt{yz}+y\sqrt{xz}+z\sqrt{xy}+\sqrt{xy}+\sqrt{yz}+\sqrt{zx}\right)\)

Suy ra cần phải chứng minh : \(\sqrt{x+yz}.\sqrt{y+zx}+\sqrt{y+zx}.\sqrt{z+xy}+\sqrt{z+xy}.\sqrt{x+yz}\ge x\sqrt{yz}+y\sqrt{xz}+z\sqrt{xy}+\sqrt{x}+\sqrt{y}+\sqrt{z}\)(*)

Thật vậy, theo bđt Bunhiacopxki ta có : \(\sqrt{x+yz}.\sqrt{y+zx}\ge\sqrt{xy}+z\sqrt{xy}\)

\(\sqrt{y+zx}.\sqrt{z+xy}\ge\sqrt{yz}+x\sqrt{yz}\)

\(\sqrt{z+xy}.\sqrt{x+yz}\ge\sqrt{xz}+y\sqrt{xz}\)

Cộng các bđt trên theo vế ta chứng minh được (*) đúng.

Vậy bđt ban đầu được chứng minh.

 

 

Bình luận (3)
Lightning Farron
14 tháng 2 2017 lúc 22:46

Ý tưởng khác

Cũng từ giả thiết suy ra \(xyz=xy+yz+xz\)

Suy ra \(\sqrt{x+yz}=\sqrt{\frac{x^2+xyz}{x}}=\sqrt{\frac{x^2+xy+yz+xz}{x}}=\sqrt{\frac{\left(x+y\right)\left(x+z\right)}{x}}\)

Theo BĐT Cauchy-Schwarz ta có \(\sqrt{\left(x+y\right)\left(x+z\right)}\ge x+\sqrt{yz}\) do đó:

\(\sqrt{x+yz}=\sqrt{\frac{\left(x+y\right)\left(x+z\right)}{x}}\ge\frac{x+\sqrt{yz}}{x}=\sqrt{x}+\sqrt{\frac{yz}{x}}\)

Tương tự cho 2 BĐT còn lại \(\sqrt{y+xz}\ge\sqrt{y}+\sqrt{\frac{xz}{y}};\sqrt{z+xy}\ge\sqrt{z}+\sqrt{\frac{xy}{z}}\)

Cộng theo vế 3 BĐT được \(VT\ge\sqrt{x}+\sqrt{\frac{yz}{x}}+\sqrt{y}+\sqrt{\frac{xz}{y}}+\sqrt{z}+\sqrt{\frac{xy}{z}}\)

\(\Leftrightarrow VT\ge\sqrt{x}+\sqrt{y}+\sqrt{z}+\frac{xy+yz+xz}{\sqrt{xyz}}\)

\(\Leftrightarrow VT\ge\sqrt{x}+\sqrt{y}+\sqrt{z}+\sqrt{xyz}\) (Đpcm)

Bình luận (0)
Trần Quang Đăng
10 tháng 9 2016 lúc 14:32

bằng 3

Bình luận (0)
Trương  Tiền  Phương
Xem chi tiết
Nyatmax
16 tháng 1 2020 lúc 20:36

\(VT\ge\frac{9}{\Sigma_{cyc}\sqrt{xy+x+y}}\ge\frac{9}{\sqrt{\left(1+1+1\right)\left(2x+2y+2z+xy+yz+zx\right)}}\ge\frac{9}{\sqrt{3\left[6+\frac{\left(x+y+z\right)^2}{3}\right]}}=\sqrt{3}\)

Bình luận (0)
 Khách vãng lai đã xóa
%Hz@
Xem chi tiết
%Hz@
Xem chi tiết
Nguyễn Hoàng Hải Dương
Xem chi tiết
Thầy Giáo Toán
19 tháng 8 2015 lúc 7:11

Bạn thiếu giả thiết \(x,y,z>0\) nhé.

Theo giả thiết \(xyz=xy+yz+zx.\)  Từ đó ta có\(\sqrt{x+yz}=\sqrt{\frac{x^2+xyz}{x}}=\sqrt{\frac{x^2+xy+yz+zx}{x}}=\sqrt{\frac{\left(x+y\right)\left(x+z\right)}{x}}.\)

Theo bất đẳng thức Bunhiacốpxki, \(\sqrt{\left(x+y\right)\left(x+z\right)}\ge x+\sqrt{yz}.\)  Do đó

\(\sqrt{x+yz}=\sqrt{\frac{\left(x+y\right)\left(x+z\right)}{x}}\ge\frac{x+\sqrt{yz}}{\sqrt{x}}=\sqrt{x}+\sqrt{\frac{yz}{x}}\), hay ta có \(\sqrt{x+yz}\ge\sqrt{x}+\sqrt{\frac{yz}{x}}.\) 

Tương tự ta có hai bất đẳng thức nữa\(\sqrt{y+zx}\ge\sqrt{y}+\sqrt{\frac{xz}{y}},\sqrt{z+xy}\ge\sqrt{z}+\sqrt{\frac{xy}{z}}\).  Cộng cả ba bất đẳng thức lại cho ta

\(\sqrt{x+yz}+\sqrt{y+zx}+\sqrt{z+xy}\ge\sqrt{x}+\sqrt{\frac{yz}{x}}+\sqrt{y}+\sqrt{\frac{zx}{y}}+\sqrt{z}+\sqrt{\frac{xy}{z}}\)

\(\Leftrightarrow\sqrt{x+yz}+\sqrt{y+zx}+\sqrt{z+xy}\ge\sqrt{x}+\sqrt{y}+\sqrt{z}+\left(\sqrt{\frac{yz}{x}}+\sqrt{\frac{zx}{y}}+\sqrt{\frac{xy}{z}}\right)\)

\(\Leftrightarrow\sqrt{x+yz}+\sqrt{y+zx}+\sqrt{z+xy}\ge\sqrt{x}+\sqrt{y}+\sqrt{z}+\frac{xy+yz+zx}{\sqrt{xyz}}\)

\(\Leftrightarrow\sqrt{x+yz}+\sqrt{y+zx}+\sqrt{z+xy}\ge\sqrt{x}+\sqrt{y}+\sqrt{z}+\sqrt{xyz}.\)    (ĐPCM)

 

Bình luận (0)
trần trúc quỳnh
18 tháng 8 2015 lúc 17:33

hừm,, dài quá nên hơi " ngán" hi hi

Bình luận (0)
%Hz@
Xem chi tiết
Nguyễn Linh Chi
29 tháng 2 2020 lúc 17:55

VT \(\ge\frac{\sqrt{3\sqrt[3]{x^3.y^3.1}}}{xy}+\frac{\sqrt{3\sqrt[3]{y^3.z^3.1}}}{yz}+\frac{\sqrt{3\sqrt[3]{z^3.x^3.1}}}{zx}\)( cauchy)

\(\sqrt{\frac{3}{xy}}+\sqrt{\frac{3}{yz}}+\sqrt{\frac{3}{zx}}\)

\(\ge3\sqrt{3}\)( cauchy)

"=" <=> x = y =z.

Bình luận (0)
 Khách vãng lai đã xóa
Trí Tiên亗
29 tháng 2 2020 lúc 18:07

Bài này dùng \(a^3+b^3\ge ab\left(a+b\right)\) được không nhỉ ??

Em ngại làm lắm cô Chi, cô thử cách này có được không ạ ?

\(xyz+x^3+y^3\ge xy\left(x+y+z\right)\)\(\Rightarrow\sqrt{1+x^3+y^3}\ge\sqrt{xy\left(x+y+z\right)}\)

Các mấy cái kia cũng biến đổi vậy.

Không chắc nx :((

Bình luận (0)
 Khách vãng lai đã xóa
l҉o҉n҉g҉ d҉z҉
Xem chi tiết
Phạm Thành Đông
2 tháng 4 2021 lúc 22:53

Đặt \(A=\sqrt{x+yz}+\sqrt{y+zx}+\sqrt{z+xy}\)

Ta có:

\(x^2+xy+yz+zx=x+xyz=x\left(x+yz\right)\)

\(\Rightarrow\frac{x\left(x+yz\right)}{x}=\frac{x^2+xy+yz+zx}{x}\)

\(\Leftrightarrow x+yz=\frac{x^2+xy+yz+zx}{x}=\frac{\left(x^2+xy\right)+\left(yz+zx\right)}{x}=\frac{\left(x+z\right)\left(x+y\right)}{x}\)

\(\Rightarrow\sqrt{x+yz}=\sqrt{\frac{\left(x+y\right)\left(x+z\right)}{x}}\)

Vì x, y, z >0 nên áp dụng bất đẳng thức Bunhiacopxki cho 2 số dương, ta được:

\(\left(x+y\right)\left(x+z\right)\ge\left(\sqrt{x^2}.+\sqrt{yz}\right)^2\)

\(\Rightarrow\sqrt{\left(x+y\right)\left(x+z\right)}\ge x+\sqrt{yz}\)

\(\Rightarrow\sqrt{\frac{\left(x+y\right)\left(x+z\right)}{x}}\ge\frac{x+\sqrt{yz}}{\sqrt{x}}\)

Do đó \(\sqrt{x+yz}\ge\frac{x+\sqrt{yz}}{\sqrt{x}}\left(1\right)\)

Chứng minh tương tự, ta được:

\(\sqrt{y+xz}\ge\frac{y+\sqrt{xz}}{\sqrt{y}}\left(2\right)\)

Chứng minh tương tự, ta được:

\(\sqrt{z+xy}\ge\frac{z+\sqrt{xy}}{\sqrt{z}}\left(3\right)\)

Từ (1), (2) và (3), ta được:

\(\sqrt{x+yz}+\sqrt{y+zx}+\sqrt{z+xy}\)\(\ge\frac{x+\sqrt{yz}}{\sqrt{x}}+\frac{y+\sqrt{zx}}{\sqrt{y}}+\frac{z+\sqrt{xy}}{\sqrt{z}}\)

\(\Leftrightarrow A\ge\sqrt{x}+\sqrt{\frac{yz}{x}}+\sqrt{y}+\sqrt{\frac{xz}{y}}+\sqrt{z}+\sqrt{\frac{xy}{z}}\)

\(\Leftrightarrow A\ge\sqrt{x}+\sqrt{y}+\sqrt{z}+\frac{yz+zx+xy}{\sqrt{xyz}}\)

 \(\Leftrightarrow A\ge\sqrt{x}+\sqrt{y}+\sqrt{z}+\frac{xyz}{\sqrt{xyz}}\)(vì \(xy+yz+zx=xyz\))

\(\Leftrightarrow A\ge\sqrt{x}+\sqrt{y}+\sqrt{z}+\sqrt{xyz}\)(điều phải chứng minh).

Dấu bằng xảy ra.

\(\Leftrightarrow\hept{\begin{cases}x=y=z>0\\xy+yz+zx=xyz\end{cases}}\Leftrightarrow x=y=z=3\)

Vậy với x, y, z là các số thực dương thỏa mãn xy + yz + zx =xyz thì:

\(\sqrt{x+yz}+\sqrt{y+zx}+\sqrt{z+xy}\ge\sqrt{x}+\sqrt{y}+\sqrt{z}+\sqrt{xyz}\).

\(\)

Bình luận (0)
 Khách vãng lai đã xóa
Hoàng Quốc Tuấn
Xem chi tiết
truongkhanhlinhbong
3 tháng 4 2020 lúc 9:53

helloo

Bình luận (0)
 Khách vãng lai đã xóa
Trần Phúc Khang
3 tháng 4 2020 lúc 14:49

Ta có \(1+x^2=x^2+xy+yz+xz=\left(x+z\right)\left(x+y\right)\)

Khi đó BĐT <=>

 \(\frac{1}{\left(x+y\right)\left(x+z\right)}+\frac{1}{\left(y+z\right)\left(x+z\right)}+\frac{1}{\left(x+y\right)\left(y+z\right)}\ge\frac{2}{3}\left(\frac{x}{\sqrt{\left(x+z\right)\left(x+y\right)}}+...\right)\)

<=> \(\frac{x+y+z}{\left(x+y\right)\left(y+z\right)\left(x+z\right)}\ge\frac{1}{3}\left(\frac{x\sqrt{y+z}+y\sqrt{x+z}+z\sqrt{x+y}}{\sqrt{\left(x+y\right)\left(y+z\right)\left(x+z\right)}}\right)^3\)

<=>\(\left(x+y+z\right)\sqrt{\left(x+y\right)\left(x+z\right)\left(y+z\right)}\ge\frac{1}{3}\left(x\sqrt{y+z}+y\sqrt{x+z}+z\sqrt{x+y}\right)^3\)

<=> \(\left(x+y+z\right)\sqrt{\left(x+y\right)\left(y+z\right)\left(x+z\right)}\ge\frac{1}{3}\left(\sqrt{x\left(1-yz\right)}+\sqrt{y\left(1-xz\right)}+\sqrt{z\left(1-xy\right)}\right)^3\)(1)

Xét \(\left(x+y\right)\left(y+z\right)\left(x+z\right)\ge\frac{8}{9}\left(x+y+z\right)\left(xy+yz+xz\right)\)

<=> \(9\left[xy\left(x+y\right)+yz\left(y+z\right)+xz\left(x+z\right)+2xyz\right]\ge8\left(xy\left(x+y\right)+xz\left(x+z\right)+yz\left(y+z\right)+3xyz\right)\)

<=> \(xy\left(y+x\right)+yz\left(y+z\right)+xz\left(x+z\right)\ge6xyz\)

<=> \(x\left(y-z\right)^2+z\left(x-y\right)^2+y\left(x-z\right)^2\ge0\)luôn đúng

Khi đó (1) <=> 

\(\left(x+y+z\right).\frac{2\sqrt{2}}{3}.\sqrt{x+y+z}\ge\frac{1}{3}\left(\sqrt{x\left(1-yz\right)}+....\right)^3\) 

<=> \(\sqrt{2\left(x+y+z\right)}\ge\sqrt{x\left(1-yz\right)}+\sqrt{y\left(1-xz\right)}+\sqrt{z\left(1-xy\right)}\)

Áp dụng buniacopxki cho vế phải ta có 

\(\sqrt{x\left(1-yz\right)}+\sqrt{y\left(1-xz\right)}+\sqrt{z\left(1-xy\right)}\le\sqrt{\left(x+y+z\right)\left(3-xy-yz-xz\right)}\)

                                                                                                       \(=\sqrt{2\left(x+y+z\right)}\)

=> BĐT được CM

Dấu bằng xảy ra khi \(x=y=z=\frac{1}{\sqrt{3}}\)

Bình luận (0)
 Khách vãng lai đã xóa